Multivariable Calculus, 7th Edition

Published by Brooks Cole
ISBN 10: 0-53849-787-4
ISBN 13: 978-0-53849-787-9

Chapter 11 - Infinite Sequences and Series - Review - Exercises - Page 803: 11

Answer

Convergent

Work Step by Step

The Comparison Test states that the p-series $\sum_{n=1}^{\infty}\frac{1}{n^{p}}$ is convergent if $p\gt 1$ and divergent if $p\leq 1$. $a_{n}=\frac{n}{n^{3}+1}\lt \frac{n}{n^{3}}=\frac{1}{n^{2}}$ Since, $\sum_{n=1}^{\infty}\frac{1}{n^{2}}$ is convergent, $\sum_{n=1}^{\infty}\frac{n}{n^{3}+1}$ is also convergent. Hence, the series $\sum_{n=1}^{\infty}\frac{n}{n^{3}+1}$ is convergent.
Update this answer!

You can help us out by revising, improving and updating this answer.

Update this answer

After you claim an answer you’ll have 24 hours to send in a draft. An editor will review the submission and either publish your submission or provide feedback.